Difference between revisions of "2011 USAJMO Problems/Problem 5"
(→Solution) |
(→Solution) |
||
Line 4: | Line 4: | ||
==Solution== | ==Solution== | ||
− | It's well known that <math>ABCD</math> is a harmonic quadrilateral. Since <math>\overline{DE} \parallel \overline{AC}</math>, we have that <math> | + | It's well known that <math>ABCD</math> is a harmonic quadrilateral. Since <math>\overline{DE} \parallel \overline{AC}</math>, we have that <math>AD=CE\rightarrow \angle ABD=\angle CBE</math>. But <math>ABCD</math> is a harmonic quadrilateral, thus <math>\overline{BD}</math> is a symmedian of triangle <math>ABC</math>, from which it follows that <math>\overline{BE}</math> is a median of <math>\triangle ABC</math>. <math>\blacksquare</math> |
== Solution 1 == | == Solution 1 == |
Revision as of 08:18, 2 October 2021
Problem
Points , , , , lie on a circle and point lies outside the circle. The given points are such that (i) lines and are tangent to , (ii) , , are collinear, and (iii) . Prove that bisects .
Solution
It's well known that is a harmonic quadrilateral. Since , we have that . But is a harmonic quadrilateral, thus is a symmedian of triangle , from which it follows that is a median of .
Solution 1
Connect segment PO, and name the interaction of PO and the circle as point M.
Since PB and PD are tangent to the circle, it's easy to see that M is the midpoint of arc BD.
∠ BOA = 1/2 arc AB + 1/2 arc CE
Since AC // DE, arc AD = arc CE,
thus, ∠ BOA = 1/2 arc AB + 1/2 arc AD = 1/2 arc BD = arc BM = ∠ BOM
Therefore, PBOM is cyclic, ∠ PFO = ∠ OBP = 90°, AF = AC (F is the interaction of BE and AC)
BE bisects AC, proof completed!
~ MVP Harry
Solution 2
Let be the center of the circle, and let be the intersection of and . Let be and be .
, ,
Thus is a cyclic quadrilateral and and so is the midpoint of chord .
~pandadude
Solution 3
This is the solution from EGMO Problem 1.43 page 242
Let be the center of the circle, and let be the midpoint of . Let denote the circle with diameter . Since , , , and all lie on .
Since quadrilateral is cyclic, . Triangles and are congruent, so , so . Because and are parallel, lies on (using Euclid's Parallel Postulate).
Solution 4
Note that by Lemma 9.9 of EGMO, is a harmonic bundle. We project through onto , Where is the point at infinity for parallel lines and . Thus, we get , and is the midpoint of . ~novus677
The problems on this page are copyrighted by the Mathematical Association of America's American Mathematics Competitions.